Đăng ký Đăng nhập
Trang chủ Số nguyên tố và đa thức bất khả quy...

Tài liệu Số nguyên tố và đa thức bất khả quy

.PDF
43
1
118

Mô tả:

.. ĐẠI HỌC THÁI NGUYÊN TRƯỜNG ĐẠI HỌC KHOA HỌC LÊ THỊ HẢI YẾN SỐ NGUYÊN TỐ VÀ ĐA THỨC BẤT KHẢ QUY LUẬN VĂN THẠC SỸ TOÁN HỌC THÁI NGUYÊN - NĂM 2014 ĐẠI HỌC THÁI NGUYÊN TRƯỜNG ĐẠI HỌC KHOA HỌC LÊ THỊ HẢI YẾN SỐ NGUYÊN TỐ VÀ ĐA THỨC BẤT KHẢ QUY LUẬN VĂN THẠC SỸ TOÁN HỌC Chuyên nghành: PHƯƠNG PHÁP TOÁN SƠ CẤP Mã số 60.46.01.13 Người hướng dẫn khoa học TS. NGUYỄN VĂN HOÀNG THÁI NGUYÊN - NĂM 2014 Mục lục Mở đầu 1 Một số kiến thức chuẩn bị 1.1 Số nguyên tố . . . . . . . 1.2 Vành đa thức . . . . . . 1.3 Đa thức bất khả quy . . 1.4 Đa thức bất khả quy trên 1.5 Đa thức bất khả quy trên 4 . . . . . . . . . . . . trường trường . . . . . . . . . . . . . . . . . . số thực và số hữu tỷ . . . . . . . . . phức . . . . . . . . . . . . . . . . . . . . . . . . . . . . . . . . . . . . . . 5 5 7 11 13 15 2 Số nguyên tố và đa thức bất khả quy 19 2.1 Liên hệ giữa số nguyên tố và đa thức bất khả quy . . . . . . 20 2.2 Đa thức bất khả quy với lũy thừa số nguyên tố . . . . . . . . 30 2.3 Ví dụ minh họa . . . . . . . . . . . . . . . . . . . . . . . . . 37 Kết luận 39 Tài liệu tham khảo 40 1 Mở đầu Sự tương tự giữa các số nguyên tố và các đa thức bất khả quy đã là một chủ đề thống trị trong sự phát triển của lý thuyết số và hình học đại số. Có các giả thuyết chỉ ra rằng mối liên hệ đó đã vượt hơn cả sự tương tự. Ví dụ, có một giả thuyết nổi tiếng của Buniakowski được phát biểu vào năm 1854: Cho đa thức f (x) hệ số nguyên thỏa mãn ba điều kiện sau i) Hệ số đầu của f (x) là dương; ii) Đa thức f (x) bất khả quy trên Q; iii) Tập các giá trị f (Z+ ) không có ước chung lớn hơn 1 khi đó đa thức f (x) nhận vô hạn các giá trị nguyên tố? (xem tài liệu S. Lang [2, Trang 323]). Một cách độc lập nó được phát biểu lại bởi Schinzel, nói về tác động của đa thức bất khả quy f (x) ∈ Z[x] (mà tập các giá trị f (Z+ ) không có ước số chung lớn hơn 1) biểu diễn vô hạn các nguyên tố. Trong trường hợp này, vấn đề dẫn đến việc quan tâm đến các số nguyên tố sinh ra từ các đa thức bất khả quy. Giả thuyết này vẫn là một trong những vấn đề lớn chưa được giải quyết trong lý thuyết số khi bậc của f lớn hơn một (Lưu ý khi f là đa thức bậc nhất, giả thuyết đó là đúng). Không khó để thấy rằng mệnh đảo của giả thuyết của Buniakowski là đúng. Một cách cụ thể hơn, nếu một đa thức biểu diễn vô hạn các số nguyên tố, thì nó là một đa thức bất khả quy. Để thấy điều này, chúng ta hãy cố gắng để phân tích ra thừa số f (x) = g(x)h(x) với g(x) và h(x) trong Z[x] có bậc dương. Thực tế, do f (x) lấy vô hạn giá trị nguyên tố, nên một trong hai g(x) hoặc h(x) nhận vô hạn giá trị ±1. Đây là một mâu thuẫn, bởi vì một đa thức có bậc dương chỉ có thể có nhận một giá trị tại hữu hạn lần. Mục đích của luận văn này là tiếp tục tìm hiểu thêm những liên hệ quan trọng giữa đa thức bất khả quy và các số nguyên tố liên quan đến giả thuyết của Buniakowski và bài toán ngược của nó như đã nêu trên. Trên cơ sở nghiên 2 cứu một số tài liệu về số nguyên tố và về đa thức bất khả quy, trong luận văn này chúng tôi lựa chọn và trình bày chi tiết lại một số tiêu chuẩn quan trọng về đa thức bất khả quy liên quan đến ứng dụng của số nguyên tố. Tài liệu tham khảo chính mà chúng tôi sử dụng là hai bài báo: • M. R. Murty, Prime numbers and irreducible polynomials, Amer. Math. Monthly 109 (2002) No. 5, 452-458 (tài liệu số [8]). • A. I. Bonciocat, N. C. Bonciocat and A. Zaharescu, On the irreducibility of polynomials that take a prime power value, Bull. Math. Soc. Sci. Math. Roumanie Tome 54 (102), No. 1 (2011), 41-54 (tài liệu số [2]). Luận văn được trình bày trong hai chương. Chương I: Một số kiến thức chuẩn bị. Nội dung của chương là trình bày tóm lược một số kiến thức cơ bản cần dùng cho chứng minh của các kết quả trong chương sau, chẳng hạn: sơ lược về số nguyên tố, về đa thức, bậc đa thức, đa thức bất khả quy, sự phân tích một đa thức thành tích các đa thức bất khả quy, một số ví dụ về đa thức bất khả quy,. . . . Chương II: Số nguyên tố và đa thức bất khả quy. Đây là chương chính của luận văn. Chương này trình bày về một số tiêu chuẩn để kiểm tra tính chất bất khả quy của một số lớp đa thức. +) Mục 2.1. Trình bày về sự liên hệ giữa số nguyên tố và đa thức bất khả quy (dựa trên bài báo [8]). Kết quả chính là định lý sau Định lý 2.1.3. Cho f (x) = am xm + am−1 xm−1 + . . . + a1 x + a0 ∈ Z[x] ai là một đa thức bậc m. Đặt H = max . Khi đó, nếu có số nguyên 0≤i≤m−1 am n ≥ H + 2 sao cho f (n) là số nguyên tố thì f (x) bất khả quy trên Z. Định lý 2.1.7 Cho b > 2 và cho p là số nguyên tố có khai triển b−adic p = an bn + an−1 bn−1 + . . . + a1 b + a0 . Khi đó f (x) = an xn + an−1 xn−1 + . . . + a1 x + a0 là bất khả quy trên Q. +) Mục 2.2. Nghiên cứu mối liên hệ giữa đa thức bất khả quy và lũy thừa số nguyên tố (dựa theo bài báo [2]). Trước hết chương này chứng minh chi tiết cho kết quả sau đây đó là một mở rộng cho Định lý 2.1.7: 3 Định lý 2.2.3. Cho p là số nguyên tố. Giả sử ps (với s ≥ 2) có biểu diễn qua hệ thống cơ số b ≥ 2 dưới dạng ps = an bn + an−1 bn−1 + . . . + a1 b + a0 , P P trong đó 0 ≤ ai ≤ b − 1 và p - ni=1 iai bi−1 . Khi đó đa thức ni=0 ai xi là bất khả quy trên Q. Phần tiếp theo của mục này là trình bày các dấu hiệu bất khả quy cho các đa thức có một hệ số nào đó đủ lớn và nó có giá trị chia hết một lũy thừa nguyên tố nào đó. Pn i Định lý 2.2.4. Cho f (x) = i=0 ai x ∈ Z[x] với a0 an 6= 0. Cho m, s, q ∈ Z (s ≥ 2) và p là số nguyên tố sao cho f (m) = ps q , p - qf 0 (m), P và |a0 | > ni=1 |ai |(|m| + |q|)i . Khi đó f (x) bất khả quy trên Q. P Định lý 2.2.5. Cho f (x) = ni=0 ai xdi ∈ Z[x], với 0 = d0 < d1 < . . . < dn và a0 a1 . . . an 6= 0. Cho m, s, q ∈ Z (với s ≥ 2), và p là số nguyên tố mà f (m) = ps q , |m| > |q| và p - qf 0 (m). Khi đó nếu có 1 ≤ j ≤ n − 1 P sao cho |aj | > (|m| + |q|)dn −dj |ai |, thì f (x) bất khả quy trên Q. i6=j Pn Định lý 2.2.6. Cho f (x) = i=0 ai xi ∈ Z[x], a0 an 6= 0. Cho f (m) = ps q với m, s, p ∈ Z, p là số nguyên tố, s ≥ 2, |m| > |q|, p - qf 0 (m) và P i−n |an | > n−1 thì f (x) bất khả quy trên Q. i=0 |ai |(|m| − |q|) Nói một cách vắn tắt thì cả ba định lý trên đều cho ta trường hợp đặc biệt khi q = 1 đó là: "Nếu f (m) là một lũy thừa của số nguyên tố (trong đó m ≥ 2 là số nguyên thỏa mãn f (m) và f 0 (m) là nguyên tố cùng nhau) và f (x) có một hệ số nào đó đủ lớn thì f (x) là đa thức bất khả quy trên Q." +) Mục 2.3 dành để trình bày một số ví dụ minh họa. Trong thời gian thực hiện luận văn này, tôi đã nhận được sự chỉ dẫn tận tình, chu đáo của Tiến sĩ Nguyễn Văn Hoàng. Tôi xin bày tỏ lòng biết ơn sâu sắc tới thầy Nguyễn Văn Hoàng đã giúp đỡ tôi hoàn thành luận văn này. Tôi xin chân thành cảm ơn Ban giám hiệu cùng các bạn đồng nghiệp trường THPT Đông Thành - Quảng Ninh đã nhiệt tình giúp đỡ tôi trong suốt quá trình học tập và hoàn thành luận văn. Tác giả 4 Chương 1 Một số kiến thức chuẩn bị Chương này nhằm mục đích trình bày lại một số kiến thức căn bản về số nguyên tố, vành đa thức, đa thức bất khả quy. Bên cạnh đó cũng trình bày một vài tiêu chuẩn bất khả quy quen biết và một số ví dụ minh họa. Những kiến thức ở chương này một phần là cần thiết cho chương sau một phần là giúp cho việc trình bày chủ đề có tính hệ thống. 1.1 Số nguyên tố Mục này ta chỉ xét trên tập các số tự nhiên N. Định nghĩa 1.1.1. Số nguyên tố là số tự nhiên lớn hơn 1 chỉ gồm có hai ước là 1 và chính nó. . Kí hiệu 1.1.2. Ký hiệu ”b|a” nghĩa là b là ước của a, ký hiệu a..b nghĩa là a chia hết cho b. Tính chất 1.1.3. i) Ước khác 1 nhỏ nhất của một số tự nhiên lớn hơn 1 là số nguyên tố. ii) Cho p là số nguyên tố, a ∈ N với a 6= 0. Khi đó (a, p) = p ⇔ p|a; (a, p) = 1 ⇔ p - a. iii) Cho a, b ∈ Z. Khi đó (a, b) = 1 nếu và chỉ nếu tồn tại x, y ∈ Z sao cho ax + by = 1. iv) Nếu tích của nhiều số chia hết cho một số nguyên tố p thì có ít nhất một thừa số chia hết cho p. 5 Chứng minh. i) Cho a là số tự nhiên > 1. Giả sử d là ước nhỏ nhất khác 1 của a. Nếu d không nguyên tố thì d = d1 d2 (với d1 , d2 > 1). Suy ra d1 |a với d1 < d, điều này mâu thuẫn với d nhỏ nhất. Vậy d là nguyên tố. ii) Nếu p = (a, p) thì hiển nhiên p|a. Ngược lại nếu p|a thì (a, p) = p. + Nếu 1 = (a, p) thì p - a (vì nếu p|a thì (a, p) = p). Ngược lại, nếu p - a thì (a, p) = 1 (vì nếu (a, p) = d > 1 thì d|p, từ đó vì p nguyên tố nên d = p, suy ra p|a, đây là điều mâu thuẫn). iii) Xét tập I = {ax + by | x, y ∈ Z}. Ta thấy I là iđêan của Z, nên tồn tại d ∈ Z, d > 0 sao cho I = dZ. Lúc đó ta dễ thấy (a, b) = d. Ngược lại, nếu (a, b) = d thì a ∈ dZ và b ∈ dZ. Từ đó I ⊆ dZ. Mặt khác, bằng thuật toán Euclid, ta tìm được x0 , y0 ∈ Z sao cho d = ax0 + by0 , suy ra d ∈ {ax + by | x, y ∈ Z} = I , do đó dZ ⊆ I . Áp dụng kết quả trên khi d = 1 ta có điều cần chứng minh. iv) Bằng quy nạp ta chỉ cần chứng minh rằng p|ab thì p|a hoặc p|b (với p là số nguyên tố, và a, b ∈ Z). Ta giả sử trái lại rằng p - a và p - b, khi đó theo ii), ta có 1 = ax + py và 1 = bx0 + py 0 với x, y, x0 , y 0 ∈ Z. Từ đó 1 = (ax + py)(bx0 + py 0 ) = abxx0 + p(axy 0 + bx0 y + pyy 0 ). Do đó ta lại áp dụng ý ii) nên ta được (ab, p) = 1, đây là điều mâu thuẫn. Tính chất 1.1.4. Tập hợp các số nguyên tố là vô hạn. Chứng minh. Giả sử chỉ có hữu hạn n số nguyên tố p1 , p2 , . . . , pn . Ta xét số tự nhiên q = p1 p2 . . . pn + 1. Rõ ràng q > 2. Gọi p là ước nhỏ nhất khác 1 của q . Khi đó p là số nguyên tố (theo Tính chất 1.1.3 i)). Mặt khác ta thấy p∈ / {p1 , . . . , pn } (vì nếu trái lại thì p|1 đó là điều mâu thuẫn). Như vậy ta lại tìm thêm được số nguyên tố nữa là p khác với mọi p1 , p2 , . . . , pn . Đó là điều mâu thuẫn với điều giả sử chỉ có n số nguyên tố p1 , . . . , pn . Ta có định lý quan trọng sau đây nói về vai trò của các số nguyên tố trong thành phần cấu tạo nên các số tự nhiên. Định lý 1.1.5. (Định lý cơ bản của số học) Mọi số tự nhiên n lớn hơn 1 đều phân tích được thành tích những thừa số nguyên tố, và sự phân tích này là duy nhất nếu không kể đến thứ tự của các thừa số. Từ đó có dạng phân tích tiêu chuẩn của một số tự nhiên n > 1 bất kỳ có dạng như sau n = pk11 pk22 . . . pkmm 6 trong đó p1 , p2 , . . . , pm là các số nguyên tố đôi một khác nhau, và k1 , k2 , . . . , km là các số tự nhiên khác 0. 1.2 Vành đa thức Mục này nhắc lại một số kiến thức căn bản về đa thức với hệ tử trên vành giao hoán A có đơn vị. Định nghĩa 1.2.1. Cho A là một vành giao hoán có đơn vị, n ∈ N, và a0 , a1 , . . . , an ∈ A. Một biểu thức có dạng f (x) = a0 + a1 x + . . . + an−1 xn−1 + an xn được gọi là một đa thức một biến x lấy hệ tử trong A. Tập tất cả các đa thức một ẩn x lấy hệ tử trên A được ký hiệu là A[x]. Nếu an 6= 0 thì ta nói bậc của f (x) là n và ký hiệu là deg(f (x)) = n; trong trường hợp này ta nói an là hệ tử cao nhất của f (x). Hai đa thức là bằng nhau nếu nó có cùng bậc và các hệ tử tương ứng là bằng nhau. Với hai đa thức f (x), g(x) ∈ A[x], khi đó tồn tại m ∈ N sao cho f (x) = Pm Pm i i i=0 bi x (lưu ý rằng không nhất thiết am 6= 0 và i=0 ai x và g(x) = bm 6= 0), ta định nghĩa tổng của f (x) và g(x) như sau: f (x) + g(x) = m X (ai + bi )xi . i=0 P P i Trong trường hợp f (x) = ni=0 ai xi và g(x) = m i=0 bi x (với n, m bất kì), ta định nghĩa tích của f (x) và g(x) bởi: f (x)g(x) = n+m X ck x k , k=0 trong đó ck = P i+j=k ai bj với mọi k = 0, . . . , n + m. Chú ý 1.2.2. Tập A[x] cùng với phép cộng và nhân các đa thức như định nghĩa trên tạo thành một vành giao hoán, ta gọi là vành đa thức một ẩn x lấy hệ tử trên A. Trong đó đa thức không trong A[x] chính là phần tử 0 của A. Phần tử 1 của A đóng vai trò phần tử đơn vị của vành A[x]. Khi A là các tập số Z, Q, R, C thì các hệ tử của các đa thức còn gọi là các hệ số. 7 Từ định nghĩa đa thức ta dễ dàng thu được một số tính chất sau đây về bậc của đa thức. Định lý 1.2.3. Giả sử f (x) và g(x) là hai đa thức khác 0 của vành A[x]. i) Nếu deg f (x) 6= deg g(x), thì ta có f (x) + g(x) 6= 0 và deg(f (x) + g(x)) = max{deg f (x), deg g(x)}. Nếu degf (x) = deg g(x) và f (x) + g(x) 6= 0 thì ta có deg(f (x) + g(x)) ≤ max{deg f (x), deg g(x)}. ii) Nếu f (x)g(x) 6= 0 thì deg(f (x)g(x)) ≤ deg f (x) + deg g(x). Chú ý 1.2.4. Giả sử A là miền nguyên. Khi đó nếu f (x), g(x) là các đa thức khác 0 của A[x] thì ta dễ thấy f (x)g(x) 6= 0. Từ đó ta thấy rằng A[x] cũng là miền nguyên. Định nghĩa 1.2.5. Cho A là một miền nguyên, và f (x), g(x) ∈ A[x] với g(x) 6= 0. Nếu tồn tại q(x) ∈ A[x] sao cho f (x) = q(x)g(x) thì ta nói rằng g(x) là ước của f (x), hay f (x) là bội của g(x), ta viết là g(x)|f (x) hoặc . f (x)..g(x) (trong trường hợp này ta cũng nói g(x) chia hết f (x), hoặc f (x) chia hết cho g(x)). Tiếp theo ta nhắc lại vài tính chất đơn giản sau đây: Bổ đề 1.2.6. Cho A là miền nguyên. Khi đó các phát biểu sau là đúng. i) Với a ∈ A và m là số tự nhiên ta có (x − a)|(xm − am ). ii) Nếu f (x) ∈ A[x] và a ∈ A thì tồn tại q(x) ∈ A[x] sao cho f (x) = (x − a)q(x) + f (a). Chứng minh. i) Kết quả được suy ra từ hằng đẳng thức xm − am = (x − a)(xm−1 + axm−2 + . . . + am−2 x + am−1 ). ii) Giả sử f (x) = an xn + an−1 xn−1 . . . + a1 x + a0 . Khi đó f (a) = an an + an−1 an−1 . . . + a1 a + a0 . Từ đó f (x) − f (a) = an (xn − an ) + an−1 (xn−1 − an−1 ) + . . . + a1 (x − a). Theo ý i), ta có xn − an , xn−1 − an−1 , . . . , x − a chia hết cho x − a. Từ đó tồn tại q(x) ∈ A[x] sao cho f (x) − f (a) = (x − a)q(x), hay f (x) = (x − a)q(x) + f (a). 8 Định nghĩa 1.2.7. Cho A là một miền nguyên và f (x) = a0 + a1 x + . . . + am xm ∈ A[x]. Giả sử K là một trường chứa A như một vành con và c ∈ K . Ta gọi c được gọi là nghiệm của f (x) nếu f (c) = a0 + a1 c + . . . + am cm = 0; trong trường hợp này ta cũng nói c là nghiệm của phương trình f (x) = 0. Bổ đề 1.2.8. Cho A là miền nguyên, f (x) ∈ A[x] và a ∈ A. Khi đó a là nghiệm của f (x) nếu và chỉ nếu f (x) là bội của x − a. Chú ý 1.2.9. Cho A là miền nguyên và f (x) ∈ A[x]. Khi đó theo bổ đề trên ta dễ dàng suy ra rằng số nghiệm của f (x) không vượt quá deg f (x). Thật vậy, giả sử deg(f (x)) = n nhưng f (x) có nghiệm là c1 , . . . , cm với m > n. Từ đó tồn tại một trường K chứa c1 , . . . , cm và nó chứa A như một vành con. Khi đó, theo bổ đề trên, suy ra tồn tại q(x) ∈ K[x] sao cho ta có biểu diễn f (x) = (x − c1 )(x − c2 ) . . . (x − cm )q(x). Từ đó dẫn đến deg(f (x)) = m + deg(q(x)) ≥ m > n, đó là điều mâu thuẫn. Bằng quy nạp người ta dễ dàng chứng minh được kết quả quan trọng sau đây, ta thường gọi là định lý phép chia với dư. Định lý 1.2.10. Cho A là miền nguyên f (x), g(x) ∈ A[x] với g(x) có hệ tử cao nhất khả nghịch trong A. Khi đó tồn tại duy nhất cặp đa thức q(x), r(x) ∈ A[x] sao cho f (x) = q(x)g(x) + r(x) trong đó r(x) = 0 hoặc deg r(x) < deg g(x). Định nghĩa 1.2.11. Cho A là miền nguyên và f (x), g(x) ∈ A[x]. Một đa thức d(x) ∈ A[x] (với d(x) 6= 0) được gọi là ước chung lớn nhất của f (x), g(x) ∈ A[x] nếu d(x)|f (x), d(x)|g(x), và nếu có h(x) ∈ A[x] mà h(x)|f (x) và h(x)|g(x) thì h(x)|d(x). Ta ký hiệu ước chung lớn nhất của f (x) và g(x) là gcd(f (x), g(x)). Nếu gcd(f (x), g(x)) = 1 thì ta nói f (x) và g(x) là nguyên tố cùng nhau. Chú ý 1.2.12. i) Nếu D là vành Gauss (miền nhân tử hóa) thì với mọi cặp đa thức f (x), g(x) ∈ D[x] không đồng thời bằng 0 ta luôn có ước chung lớn nhất của chúng, tức là tồn tại gcd(f (x), g(x)). Ước chung lớn nhất của hai đa thức (nếu có) là duy nhất sai khác một phần tử khả nghịch của D. ii) Như vậy, với mọi f (x), g(x) ∈ Z[x] không đồng thời bằng 0, luôn tồn tại ước chung lớn nhất gcd(f (x), g(x)). 9 Trong trường hợp xét các đa thức trên một trường ta còn có tính chất sau. Mệnh đề 1.2.13. Nếu K là trường và f (x), g(x) ∈ K[x] là hai đa thức không đồng thời bằng 0, thì luôn tồn tại gcd(f (x), g(x)) và tồn tại a(x), b(x) ∈ K[x] sao cho gcd(f (x), g(x)) = a(x)f (x) + b(x)g(x). Chứng minh. Ta có K[x] là vành chính. Xét iđêan I = {a(x)f (x)+b(x)g(x) | a(x), b(x) ∈ K[x]}. Vì I là iđêan chính nên có d(x) ∈ I sao cho I = hd(x)i. Khi đó d(x) = gcd(f (x), g(x)). Hệ quả 1.2.14. Cho p(x), f (x), g(x) ∈ K[x]. Nếu gcd(p(x), f (x)) = 1 và p(x)|f (x)g(x) thì p(x)|g(x). Chứng minh. Theo giả thiết, tồn tại a(x), b(x) ∈ K[x] sao cho 1 = p(x)a(x)+ f (x)b(x). Suy ra g(x) = p(x)a(x)g(x) + f (x)b(x)g(x). Do p(x) là ước của đa thức ở vế phải nên p(x)|g(x). Kí hiệu 1.2.15. Cho K là trường. Với 0 6= g(x) ∈ K[x], ta kí hiệu g ∗ (x) = g(x)/an (trong đó an là hệ số cao nhất của g(x)). Chú ý rằng g ∗ (x) là đa thức có hệ số cao nhất là 1. Để tìm ước chung lớn nhất ta có thuật toán sau. Mệnh đề 1.2.16. (Thuật toán Euclid tìm ước chung lớn nhất). Cho K là trường. Cho hai đa thức f (x), g(x) ∈ K[x] với g(x) 6= 0. Nếu g(x)|f (x) thì gcd(f (x), g(x)) = g ∗ (x). Nếu ngược lại, chia liên tiếp ta được f (x) = q(x)g(x) + r(x), r(x) 6= 0, deg r(x) < deg g(x). g(x) = q1 (x)r(x) + r1 (x), r1 (x) 6= 0, deg r1 (x) < deg r(x). ... rn−2 (x) = qn (x)rn−1 (x) + rn (x), rn (x) 6= 0, deg rn (x) < deg rn−1 (x). rn−1 (x) = qn+1 (x)rn (x). Khi đó gcd(f (x), g(x)) = rn∗ (x). Chứng minh. Từ đẳng thức cuối ta có rn (x)|rn−1 (x). Thay vào đẳng thức thứ hai từ dưới lên ta có rn (x)|rn−2 (x). Tiếp tục lập luận với các đẳng thức từ dưới lên trên ta suy ra rn (x)|g(x) và rn (x)|f (x). Do đó rn∗ (x)|f (x) và rn∗ (x)|g(x). 10 Giả sử h(x)|f (x) và h(x)|g(x). Từ đẳng thức đầu tiên ta có h(x)|r(x). Từ đẳng thức thứ hai ta có h(x)|r1 (x). Cứ tiếp tục lập luận trên với các đẳng thức từ trên xuống dưới ta có h(x)|rn (x). Do đó h(x)|rn∗ (x). 1.3 Đa thức bất khả quy Định nghĩa 1.3.1. Cho A là một miền nguyên. Một đa thức f (x) ∈ A[x] được gọi là bất khả quy trên A nếu f (x) 6= 0, f (x) không khả nghich và f (x) không có ước thực sự. Một đa thức khác 0, không khả nghịch, mà không bất khả quy thì được gọi là đa thức khả quy. Chú ý 1.3.2. i) Cho f (x) ∈ A[x] (với A là miền nguyên). Ta nói f (x) không có ước thực sự, nghĩa là, giả sử có g(x), h(x) ∈ A[x] sao cho f (x) = g(x)h(x) thì kéo theo g(x)|1 hoặc h(x)|1. ii) Khi A được thay thế bởi trường K , ta có khái niệm đa thức bất khả quy như sau: Ta nói đa thức f (x) ∈ K[x] là bất khả quy trên K nếu deg(f (x)) > 0 và f (x) không phân tích được thành tích hai đa thức có bậc nhỏ hơn. Sau đây là một số ví dụ về đa thức bất khả quy. Bổ đề 1.3.3. Cho K là trường. Các phát biểu sau là đúng. i) Đa thức bậc nhất trong K[x] luôn bất khả quy. ii) Nếu f (x) bậc lớn hơn 1 và có nghiệm trong K thì f (x) khả quy. iii) Đa thức bậc 2 và bậc 3 trong K[x] là bất khả quy nếu và chỉ nếu nó không có nghiệm trong K . iv) Đa thức f (x) ∈ K[x] có bậc dương là bất khả quy nếu và chỉ nếu f (x + a) là bất khả quy với mọi a ∈ K . Chứng minh. i) Rõ ràng đa thức bậc nhất không thể là tích của hai đa thức bậc bé hơn. ii) Nếu deg f (x) > 1 và f (x) có nghiệm x = a ∈ K thì f (x) = (x − a)g(x) trong đó deg g(x) = deg f (x) − 1 ≥ 1. Vì thế f (x) khả quy. iii) Cho f (x) có bậc 2 hoặc 3. Nếu f (x) khả quy thì nó phân tích được thành tích của hai đa thức bậc thấp hơn, một trong hai đa thức đó phải có bậc 1, 11 do đó f (x) có nghiệm trong K . Nếu f (x) có nghiệm trong K thì theo ii), f (x) khả quy. iv) Cho đa thức f (x) ∈ K[x] có bậc dương và a ∈ K . Với mỗi h(x) ∈ K[x], ta đặt h1 (x) = h(x − a). Lưu ý rằng deg h1 (x) = deg h(x) với mọi h(x) ∈ K[x]. Vì thế nếu f (x + a) = k(x)g(x) là phân tích của f (x + a) thành hai đa thức có bậc thấp hơn khi và chỉ khi f (x) = k1 (x)g1 (x) là phân tích của f (x) thành tích của hai đa thức có bậc thấp hơn. Vì vậy f (x) bất khả quy khi và chỉ khi f (x + a) bất khả quy. Định nghĩa 1.3.4. Cho K là một trường chứa trường T như một trường con và a ∈ K . Ta nói a là phần tử đại số trên T nếu tồn tại một đa thức 0 6= f (x) ∈ T [x] nhận a làm nghiệm. Nếu a không đại số trên T thì ta nói a là siêu việt trên T . Mệnh đề 1.3.5. Cho K là một trường chứa trường T như một trường con và a ∈ K là phần tử đại số trên T . Khi đó tồn tại duy nhất một đa thức p∗ (x) ∈ T [x] bất khả quy có hệ số cao nhất là 1 và nó nhận a làm nghiệm, và mọi đa thức g(x) ∈ T [x] nhận a làm nghiệm đều là bội của p∗ (x). Chứng minh. Vì a là nghiệm của một đa thức khác 0 với hệ số trong T nên tồn tại đa thức p(x) khác 0 với hệ số trong T có bậc bé nhất nhận a làm nghiệm. Ta chứng minh p∗ (x) bất khả quy (với p∗ (x) = p(x)/b trong đó b là hệ số cao nhất của p(x)). Giả sử p∗ (x) không bất khả quy. Khi đó p∗ (x) phân tích được thành tích của hai đa thức trong T [x] có bậc thấp hơn, và do đó một trong hai đa thức này phải nhận a làm nghiệm, điều này là mâu thuẫn với cách chọn p(x). Giả sử g(x) ∈ T [x] nhận a làm nghiệm. Nếu p∗ (x) không là ước của g(x) thì vì p∗ (x) bất khả quy nên gcd(g(x), p∗ (x)) = 1, do đó 1 = p∗ (x)q(x) + g(x)h(x) với q(x), h(x) ∈ T [x]. Thay x = a vào cả hai vế ta được 1 = 0, điều này vô lý. Vậy g(x) chia hết cho p∗ (x). Giả sử q(x) ∈ G[x] cũng là đa thức bất khả quy có hệ số cao nhất là 1, nhận a làm nghiệm. Theo chứng minh trên, q(x) là bội của p∗ (x). Viết q(x) = p∗ (x)k(x). Vì q(x) bất khả quy nên k(x) = b ∈ K . Do đó q(x) = bp∗ (x). Đồng nhất hệ số cao nhất của hai vế với chú ý rằng q(x) và p∗ (x) đều có hệ số cao nhất là 1, ta suy ra b = 1. Vì thế p∗ (x) = q(x). 12 Định nghĩa 1.3.6. Đa thức p∗ (x) ∈ T [x] bất khả quy có hệ số cao nhất là 1 xác định như trong Mệnh đề 1.3.5 được gọi là đa thức tối tiểu của a (hoặc đa thức bất khả quy của a). √ Ví dụ 1.3.7. 1) Đa thức x3 − 5 ∈ Q[x] là đa thức bất khả quy của 3 5 ∈ R. 2) Đa thức x2 + 1 ∈ R[x] là đa thức bất khả quy của i ∈ C. Định lý 1.3.8. Có vô số đa thức một ẩn với hệ số cao nhất là 1 bất khả quy trên trường K . Chứng minh. Nếu K là trường vô hạn thì các đa thức dạng x − a với a ∈ K là các đa thức với hệ số cao nhất là 1 bất khả quy trên K. Có vô số đa thức như vậy. Trong trường hợp K là trường hữu hạn, giả sử chỉ có n đa thức bất khả quy p1 (x), p2 (x), . . . , pn (x) với hệ số cao nhất là 1. Khi đó đa thức f (x) = p1 (x)p2 (x) . . . pn (x) + 1 có ít nhất một ước bất khả quy với hệ số cao nhất là 1 vì deg f (x) ≥ n. Ước đó phải khác p1 (x), p2 (x), . . . , pn (x) vì nếu không nó sẽ là ước của f (x) − p1 (x)p2 (x) . . . pn (x) = 1, điều này vô lý. Vậy K[x] phải có vô hạn đa thức bất khả quy với hệ số cao nhất là 1. 1.4 Đa thức bất khả quy trên trường số thực và phức Trước hết ta nhắc lại mà không chứng minh một kết quả quan trọng sau đây, nó được gọi là định lý cơ bản của đại số. Định lý 1.4.1. Mọi đa thức f (x) bậc n ≥ 1 trên trường số phức đều có n nghiệm phức (kể cả số bội). Định lý này có các hệ quả sau đây. Hệ quả 1.4.2. Các đa thức bất khả quy của vành C[x] (với C là trường số phức) chỉ là các đa thức bậc nhất. 13 Chứng minh. Hiển nhiên các đa thức bậc nhất là các đa thức bất khả quy. Giả sử f (x) là một đa thức của C[x] có bậc lớn hơn 1. Theo Định lý 1.4.1, f (x) có nghiệm phức c ∈ C. Vậy f (x) chia hết cho x − c, do đó f (x) không bất khả quy. Mệnh đề 1.4.3. Nếu một số phức α là nghiệm của đa thức f (x) với hệ số thực thì số phức liên hợp α cũng là một nghiệm của f (x). Chứng minh. Giả sử f (x) = an xn + an−1 xn−1 + . . . + a1 x + a0 là một đa thức với hệ số thực và α là một nghiệm phức của f (x). Khi đó an αn + an−1 αn−1 + . . . + a1 α + a0 = 0. Lấy liên hợp hai vế của đẳng thức trên ta được an αn + an−1 αn−1 + . . . + a1 α + a0 = 0. Điều này chứng tỏ số phức liên hợp α cũng là nghiệm của f (x). Định lý 1.4.4. Các đa thức bất khả quy trong R[x] (với R là trường số thực) chỉ là các đa thức bậc nhất và các đa thức bậc hai ax2 + bx + c với biệt số ∆ = b2 − 4ac < 0. Chứng minh. Dễ dàng thấy rằng các đa thức bậc nhất và đa thức bậc hai với biệt số ∆ < 0 là các đa thức bất khả quy trên R. Ta chứng minh chiều ngược lại. Giả sử f (x) là đa thức bất khả quy trên R và α là một nghiệm phức. Nếu α ∈ R thì f (x) chia hết cho x − α, do f (x) bất khả quy nên f (x) = k(x − α), k ∈ R∗ , vậy f (x) là đa thức bậc nhất. Nếu α ∈ C \ R thì α cũng là nghiệm của f (x), do đó f (x) chia hết cho p(x) = (x − α)(x − α) = x2 − (α + α)x + αα. p(x) là một tam thức bậc hai với hệ số thực và có biệt số ∆ < 0. Do f (x) bất khả quy nên f (x) = kp(x), k ∈ R∗ . Vậy f (x) là tam thức bậc hai với biệt số ∆ < 0. 14 1.5 Đa thức bất khả quy trên trường số hữu tỷ Trong vành Q[x] các đa thức trên trường hữu tỷ thì vấn đề phức tạp hơn nhiều. Sau đây ta sẽ trình bày tiêu chuẩn Eisenstein là một điều kiện đủ để nhận biết một đa thức là bất khả quy trên Q. Để chuẩn bị chứng minh tiêu chuẩn ấy, ta cần một vài khái niệm và bổ đề sau đây: Định nghĩa 1.5.1. Giả sử f (x) là một đa thức với hệ số nguyên, f (x) gọi là nguyên bản nếu các hệ số của f (x) không có ước chung nào khác ngoài ±1. Bổ đề 1.5.2. Tích của hai đa thức nguyên bản là một đa thức nguyên bản. Chứng minh. Giả sử f (x) = a0 + a1 x + . . . + am xm và g(x) = b0 + b1 x + . . . + bn xn là hai đa thức nguyên bản. Ta chỉ cần chứng minh rằng cho một số nguyên tố p tùy ý, p không chia hết các hệ số của đa thức tích f (x)g(x). Rõ ràng p không chia hết đồng thời mọi hệ số của f (x) và g(x). Giả sử p chia hết a0 , . . . , ar−1 , b0 , . . . , bs−1 và p không chia hết ar và bs . Ta xét hệ số cr+s của đa thức tích f (x)g(x). Ta có cr+s = (. . . + ar−1 bs+1 ) + ar bs + (ar+1 bs−1 + . . .) trong đó p chia hết các tổng trong các dấu ngoặc, nhưng không chia hết tích ar bs vì p là nguyên tố. Do đó p không chia hết cr+s Bổ đề 1.5.3. Nếu f (x) là một đa thức với hệ số nguyên có bậc lớn hơn 0 và f (x) không bất khả quy trong Q[x], thì f (x) phân tích được thành một tích những đa thức bậc khác 0 với hệ số nguyên Chứng minh. Giả sử f (x) không bất khả quy trong Q[x], thì f (x) có thể viết f (x) = ϕ(x)ψ(x) với ϕ(x) và ψ(x) là những ước thực sự của f (x) trong Q[x]. Ta có thể viết a c ϕ(x) = g(x), ψ(x) = h(x) b d trong đó g(x), h(x) là những đa thức nguyên bản và a, b, c, d là những số nguyên. Do đó p f (x) = g(x)h(x) q 15 p ac = và p, q nguyên tố cùng nhau. Ta kí hiệu các hệ số của đa thức q bd tích g(x)h(x) bằng ei , thế thì theo Bổ đề 1.5.2, g(x)h(x) là nguyên bản, cho nên các ei không có ước chung nào khác ngoài ±1. Mặt khác vì f (x) ∈ Z[x] pei nên các số phải là nguyên. Do đó q chia hết mọi ei vì q nguyên tố với q p. Ta suy ra q = ±1, tức là f (x) = ±p g(x)h(x). Vì ϕ(x) và ψ(x) là những ước thực sự của f (x) trong Q[x], nên g(x) và h(x) là những đa thức bậc 0 của Z[x]. với Chú ý 1.5.4. Cho f (x) ∈ Q[x] khi đó ta có thể viết f (x) = ab g(x) trong đó g(x) là đa thức hệ số nguyên và a/b là số hữu tỷ khác 0. Do a/b khả nghịch trong Q nên tính bất khả quy trong Q[x] của f (x) và g(x) là như nhau. Do đó về sau để xét tính bất khả quy trong Q[x] người ta chỉ cần quan tâm đến việc xét tính bất khả quy trong Q[x] của các đa thức hệ số nguyên. Dưới đây là một tiêu chuẩn quan trọng để kiểm tra tính bất khả quy của một đa thức. Định lý 1.5.5. (Tiêu chuẩn Eisenstein) Giả sử f (x) = a0 + a1 x + . . . + an xn (n > 1) là một đa thức với hệ số nguyên, và giả sử có một số nguyên tố p sao cho p không chia hết hệ số cao nhất an , nhưng p chia hết các hệ số còn lại và p2 không chia hết hệ số tự do a0 . Thế thì đa thức f (x) là bất khả quy trong Q[x] Chứng minh. Giả sử f (x) có những ước thực sự trong Q[x]. Theo Bổ đề 1.5.3, f (x) có thể viết thành f (x) = g(x)h(x), trong đó g(x) = b0 + b1 x + . . . + br xr , bi ∈ Z, 0 r ≥ k > 0) là hệ số đầu tiên của g(x) không chia hết cho p (bởi vì nếu không thì mọi hệ số của g(x) đều chia hết cho p, dẫn đến an = br cs chia hết cho p, điều này trái giả thiết). Ta hãy xét hệ số ak = bk c0 + bk−1 c1 + . . . + b0 ck , trong đó ak , bk−1 , . . . , b0 đều chia hết cho p. Vậy bk c0 phải chia hết cho p. Vì p là nguyên tố, ta suy ra bk chia hết cho p, hoặc c0 chia hết cho p, mâu thuẫn với giả thiết về bk và c0 . Ví dụ 1.5.6. 1) Đa thức x4 + 6x3 − 18x2 + 42x + 12 là bất khả quy trong Q[x]. Thật vậy, ta có thể áp dụng tiêu chuẩn Eisenstein với p = 3. 2) Đa thức xn + pxn−1 + pxn−2 + . . . + p với p là một số nguyên tố tùy ý, là bất khả quy trong Q[x]. Thật vậy, ta có thể áp dụng tiêu chuẩn Eisenstein với số nguyên tố p. Định lý 1.5.7. (Tiêu chuẩn Osada) Cho đa thức hệ số nguyên f (x) = xn + a1 xn−1 + . . . + an−1 x ± p trong đó p là số nguyên tố và n ≥ 2. Nếu p > 1 + |a1 | + . . . + |an−1 | thì f (x) là bất khả quy trong Q[x]. Chứng minh. Giả sử f (x) là khả quy trong Q[x]. Khi đó, theo Bổ đề 1.5.3 ta có f (x) = g(x)h(x), trong đó g(x), h(x) là các đa thức bậc dương với các hệ số nguyên. Vì p là số nguyên tố nên một trong các số hạng tự do của g(x) hay h(x) phải bằng ±1, chẳng hạn hệ số tự do của g(x) bằng ±1. Vậy giá trị tuyệt đối của tích các nghiệm của g(x) phải bằng 1. Khi đó, phương trình g(x) = 0 phải có một nghiệm phức α với |α| ≤ 1. Vì α cũng là nghiệm của f (x) = 0 nên p = |αn + a1 αn−1 + . . . + an−1 α| ≤ 1 + |a1 | + . . . + |an−1 |, đây là điều mâu thuẫn. Vậy chứng tỏ f (x) là bất khả quy trong Q[x]. 17 Ví dụ 1.5.8. Cho số tự nhiên n ≥ 2. Khi đó đa thức q(x) = xn − 18xn−1 + 3x2 − 2011 luôn luôn là bất khả quy trong Q[x]. Thật vậy, vì 2017 là số nguyên tố và 2017 > 1 + 18 + 3 nên q(x) là bất khả quy theo Định lý 1.5.7. Định lý 1.5.9. (Tiêu  chuẩn Polya) Cho f (x) là đa thức bậc n với các hệ n+1 số nguyên. Đặt m = . Giả sử cho n số nguyên khác nhau d1 , . . . , dn 2 m! sao cho |f (di )| < m và các số di đều không là nghiệm của f (x). Khi đó 2 f (x) là bất khả quy. Chứng minh. Giả sử f (x) là một đa thức khả quy. Khi đó, theo Bổ đề 1.5.3, ta có biểu diễn f (x) = g(x)h(x) với g(x), h(x) là những đa thức bậc dương hệ số nguyên. Hiển nhiên deg(g), deg(h) < n. Không mất tính tổng quát ta có thể giả thiết deg(h) ≤ deg(g) = s. Ta có m ≤ s < n. Ta thấy ngay g(di ) 6= 0 và g(di ) chia hết f (di ). Do đó |g(di )| ≤ |f (di )| < m! 2m s! với mọi i. Mặt khác, vì deg(g) = s, nên tồn tại i để g(di )| ≥ s (theo Polya). 2 s! m! m! s! Vì s ≥ m nên s ≥ m . Vậy m > |g(di )| ≥ s , đây là điều mâu thuẫn. 2 2 2 2 Điều đó chỉ ra rằng f (x) là một đa thức bất khả quy. 18
- Xem thêm -

Tài liệu liên quan

Tài liệu xem nhiều nhất